PT3.S2.Q18 - Ian Ragnall's article

cgc222-1-1-1cgc222-1-1-1 Alum Member
edited June 2022 in Logical Reasoning 8 karma

I got tricked. I did not pay enough attention to the words. I chose option answer E, but it says "most articles" whereas C has "Ragnall's" which is more specific.

Admin Note: Edited title. Please use the format: "PT#.S#.Q# - brief description of the question"

Comments

  • Avatar RokuAvatar Roku Member
    71 karma

    Hi there,

    Yes, the correct answer choice is (C). I think you may have been tricked by (E) by inferring that "number of other articles" concluded that financial problems are a major problem in divorces. However, they never concluded that. Only Ragnall concluded that. The relationship between Ragnall and the many other articles isn't that they concluded that money is a major problem. The similarity between them is that they simply relied on data that showed people choosing "money" as a major problem in divorces.

    (C) is correct because the conclusion here says "The survey data do not establish that financial problems are the major problem in contemporary marriages." This is referring back to Ragnall's conclusion. This conclusion, translated, is basically saying "Nah Ragnall, you can't draw your conclusion based off the data. I'm not saying that your conclusion is wrong or false. I'm just saying that you can't draw your conclusion, because couples often express other types of marital frustrations in financial terms."

    Moreover, this is somewhat a good argument. At first I though this was a unjustified/ reject flaw, which is when the author asserts that the other person's claim is unjustified, and then concludes the opposite or deems the other person's claim false. But, in this stimulus, the author is merely pointing to another possible "explanation," and drawing a relatively weak conclusion saying that "I'm not saying your conclusion is false or wrong. I'm just saying that because of this possible explanation, your conclusion can't be justified by the data you used."

    I really hope this helped and didn't confuse you. At first when I read the stimulus and read the answer choices, I chose (C). But then my answer key said (E) which confused the hell out of me, until I realized that I was reading the wrong answer key!

  • dpgold24dpgold24 Core Member
    32 karma

    ^This was super helpful to me! I picked AC E on both first pass & BR, and felt pretty confident because the stim started with the qualifier of "like a number of other articles"...which made me think E held water when it said "Many". My reasoning for E was that because the stim had that qualifier, many was appropriate, otherwise it went too far.

    Above comment is super helpful in pointing out that we don't know if those many other articles draw that conclusion. All we know is that they use the same study as R, but he is the one that draws the conclusion, but the evidence doesn't add up. Therefore, (C) is right, his conclusion is not adequately justified bc it relies on that study.

Sign In or Register to comment.